June 2001 Pg. 206 16 and 26 Forum

Prepare for the LSAT or discuss it with others in this forum.
Post Reply
Bigsby

New
Posts: 71
Joined: Sat Jan 15, 2011 2:03 pm

June 2001 Pg. 206 16 and 26

Post by Bigsby » Wed Apr 06, 2011 6:16 pm

16. In a recent judicial decision, a contractor was ordered to make restitution to a company because of a bungled construction job, even though the company had signed a written agreement prior to entering into the contract that the contractor would not e financially liable should the task not be adequately performed. Thus, it was morally wrong for the company to change its mind and seek restitution.

Which one of the following principles, if valid, most helps to justify the ethicist's reasoning?

Choice B says- It is morally wrong to seek a penalty for an action for which the agent is unable to make restitution.

This is incorrect but I was wondering what it was saying exactly and why it is incorrect?

And this question:

26. The media now devote more coverage to crime than they did ten years ago. Yet this is not because the crime rate has increased, but rather because the public is now more interested in reading and hearing about crime. After all, a crucial factor in the media's decisions about what issues to cover and to what extent to cover them in the interests of their audiences.

the question is:

The proposition that the public is now more interested in reading and hearing about crime plays which one of the following roles in the argument?

A- It supports the conclusion that the media now devote more coverage to crime than the crime rate alone justifies.

Why couldn't this be correct? The answer is E- It is offered as an alternative explanation of why the media devote more coverage to crime now than they did ten years ago.

I see why that is correct but why doesn't the line support the conclusion as well?

User avatar
Jeffort

Gold
Posts: 1888
Joined: Wed Jun 18, 2008 4:43 pm

Re: June 2001 Pg. 206 16 and 26

Post by Jeffort » Wed Apr 06, 2011 10:33 pm

For #16:

Answer choice (B) is wrong because the argument does not contain facts that you can apply the principle to. The premises of the argument give ZERO facts about the contractors ability/inability to make restitution, making that principle useless for adding support to the conclusion.

#26:

(A) is wrong because the part the stem is asking about IS the conclusion. The first sentence of the argument that (A) references (in a distorted way) is a premise, not the conclusion.

Post Reply

Return to “LSAT Prep and Discussion Forum”